1.Waheeda mixes water with some lemon juice to make lemonade. Write an equation to represent how much lemon juice is needed when Waheeda uses 18 ounces of water.

2. Identify the independent and dependent variables in the situation.
Independent variable: Dependent variable: b) Write an equation representing the amount Waheeda earns in relation to the number of glasses of lemonade she sells.
Equation: c) In which Quadrant of a graph would her data appear? Explain.

1.Waheeda Mixes Water With Some Lemon Juice To Make Lemonade. Write An Equation To Represent How Much

Answers

Answer 1

1. The equation is 18 = 3 × x

2. a) The independent variable is the of ounces of lemon juice in the lemonade

b) A = a × b

c) First quadrant

The procedure to find the above answers are presented here as follows;

1. The following is the table of values of the data

[tex]\begin{array}{ccc} Lemon \ juice \ ounces \ (x)& & Water \ in \ ounces \ (y)\\2&&6\\3&&9\\4&&12\\5&&15\end{array}[/tex]

The given data in the table shows that the Lemon juice ounces, x, and the Water in ounces, y, have a direct proportionality relationship which is given as follows;

y ∝ x

∴ y = k × x

Where;

k = The constant of proportionality

k = y/x

Therefore;

k = 6/2 = 9/3 = 12/4 = 15/5 = 3

k = 3

y = k × x

∴ y = 3 × x

Therefore;

The equation to represent how much lemon juice, x, is needed when Waheeda uses y = 18 ounces of water  is given by placing y = 18 in y = 3 × x as to get;

18 = 3 × x

∴ x = 18/3 = 6; 6 ounces of lemon juice is needed with 18 ounces of water

2. The independent variable is the input or causing variable that determines the output variable

Therefore, the independent variable is the number of ounces of lemon juice required to be mixed with a given number of ounce of water to make lemonade

b) Let a represent the selling price for a glass of lemonade, let b represent the number of glasses of lemonade she sells, and let A represent the amount she earns, we have;

A = a × b

c) The given x and y values in the data are both positive values, which indicate that the data would appear in the first quadrant

Learn more about dependent and independent variable here;

https://brainly.com/question/18456313


Related Questions

Adams Company revenues are $500 on invested capital of $250. Expenses are currently 60% of sales. If Angelo Company can reduce its capital investment by 20% in Adams Company, return on investment will be _____.

Answers

Answer:

100%

Step-by-step explanation:

The formula to calculate the return on investment is:

ROI=(Net Profit/Total Investment)*100

Net profit=Revenues-expenses=500-(500*0.6)=500-300=200

Total investment=250-(250*0.2)=250-50=200

Now, you can replace the values:

ROI= (200/200)*100

ROI= 100%

According to this, the answer is that If Angelo Company can reduce its capital investment by 20% in Adams Company, return on investment will be 100%.

the work in an office takes 180 hours to complete every work
each person in the office works for 35 hours a week
what is the smallest number of people needed to complete the work?​

Answers

Answer:

Minimum People required = 5

Step-by-step explanation:

Total hours required to complete the work every week = 150 hrs.

Number of hours worked per week by one person = 32 hr

∴ Number of people required to complete the work per week = Total number of hrs to complete the work ÷ No of hrs work per person

∴ Number of people = 150 ÷ 32

∴ Number of people = 4.6875

This is the minimum number of people. But no of people cannot be a fraction.

Thus, rounding the number to next integer.

∴ Smallest number of people needed to complete the work = 5

1/5√75 -10√1/2+√125-2√1/2

Answers

Answer:6.91239069507

Step-by-step explanation:

Solve each system of equations 4x+6y=3 and -10x-15y=-4

Answers

Answer:

There is no solution

Step-by-step explanation:

they all subtract eachother out

Find the constant of proportionality (r) in the equation y = r x

Answers

Answer:

r = 11

Step-by-step explanation:

y = r x

r is the constant of proportionality

To find r pick any values of x and y provided and substitute it into the above formula and solve for r.

That's

using

x = 2

y = 22

We have

22 = 2r

Divide both sides by 2

r = 11

Therefore the constant of proportionality is 11

Hope this helps you

Bryan decides he wants to help pay for a birthday party for his little brother at the ice rink. It cost $50 to rent the party room and then $4 for each person attending. Bryan only has $100 to spend at the party. a) What are the constraints for this situation? b) Find the domain and range for this situation. Make sure you include all values for each using correct notation.

Answers

Answer:

a) 4*x + 50 ≤ 100  

b) Domain   x (0 ; 12 )     Range   f(x)   ( 50  ; 98 )

Step-by-step explanation:

The constraint is:

4*x + 50 ≤ 100                   where "x" is the number of persons

b) Domain for x

x = 0    up to  x = 12           x (0 ; 12 )

c) Range for f(x)

f(x) = 4*x + 50

f(0)  = 4*0 + 50      f(0) =  50

f(12) = 4*12 + 50    f (12) =  98

f(x)   ( 50  ; 98 )

Three whole numbers have an HCF of 3 and an LCM of 180. Two of the numbers are 45 and 60. Find the third number.

Answers

Answer:

Step-by-step explanation:

45=3×3×5

60=2×2×3×5

L.C.M=180

2| 180

2|90

3|45

3|15

3|5

180=2×2×3×3×5

third number=2×3=6

or 2×2×3=12

or2×3×3=18

or 2×2×3×3=36

so third number can be one of  6,12,18,36

The Stem-and-Leaf Graph shows the amount of money each student spends on food per day in dollars. What is the median for the data in this Stem-and-Leaf Plot? A. $55 B. $73 C. $81 D. $84

Answers

Answer:

B) $73

Step-by-step explanation:

add all of your values and divide by the amount of values

52+55+55+55+59+64+66+68+72+73+73+73+73+75+81+81+83+84+84+86+87=

1,499

1,499 divided by 21 = 71.3809523...

which rounds to 73

HOPE THIS HELPS!!! :)

The median is of $48 in the stem leaf plot and option B is correct.

What is Statistics?

Statistics is the discipline that concerns the collection, organization, analysis, interpretation, and presentation of data.

The median of a data-set is the value that separates the bottom 50% from the upper 50% of values.

The graph has 16 values, already ordered.

It is an even number, hence the median is the mean of the 8th and the 9th values, which considering the key are both 48,

Hence, the median is of $48 and option B is correct.

To learn more on Statistics click:

https://brainly.com/question/30218856

#SPJ3

Stem-and-Leaf Plot shows the amount of money each student spends

on travel per day in dollars. What is the median for the data in this graph?

Stem Leaf

A) $35

B) $48

C) $53

D) $54

Find m∠A.
A. 32
B. 26
C. 27
D. 30

Answers

Answer: B. 26

============================================

Use law of cosines to find angle A

a^2 = b^2 + c^2 - 2*b*c*cos(A)

9^2 = 6^2 + 14^2 - 2*6*14*cos(A)

81 = 36 + 196 - 168*cos(A)

81 = 232 - 168*cos(A)

81 - 232  = -168*cos(A)

-151  = -168*cos(A)

-168*cos(A) = -151

cos(A) = (-151)/(-168)

cos(A) = 0.8988095

A = arccos(0.8988095)

A = 25.9979801

A = 26 degrees approximately

The angle m∠A will be 27. so option C is correct.

What is a cosine law?

Cosine law is a formula relating the length of the sides of a triangle to the cosine of one angle of the triangle.

u² = s² + t² - 2(s)(t)·cos U

Use law of cosines to find angle A

a^2 = b^2 + c^2 - 2*b*c*cos(A)

9^2 = 6^2 + 14^2 - 2*6*14*cos(A)

81 = 36 + 196 - 168*cos(A)

81 = 232 - 168*cos(A)

81 - 232  = -168*cos(A)

-151  = -168*cos(A)

-168*cos(A) = -151

cos(A) = (-151)/(-168)

cos(A) = 0.8988095

A = 25.9979801

A = 26 degrees approximately

Therefore, the angle m∠A will be 27. so option C is correct.

Learn more about sine to cosine conversion here:

https://brainly.com/question/1421592

#SPJ5

HELP NOW - Please help me
100 POINTS

Answers

Answer:

SAS

43ft

Step-by-step explanation:

We know that two sides are equal

PQ = ST and QR = TU and the angle between them is equal Q = T

We can use the SAS (side angle side)

Since the triangles are congruent

PR = SU

6y+5 = 8y

Subtract 6y from each side

6y+5-6y = 8y-6y

5 = 2y

Divide by 2

5/2 = y

2.5 = y

PR = ST = 8y = 8(2.5) =20

The perimeter is 9+14+20 = 43

Answer:

B. =35ft

Step-by-step explanation:

The perimeter of a right angled triangle is a+b+c

which is 9+4+6y+5 =90

collect like terms

9+4+5+6y=90

18 + 6y = 90

6y = 90 - 18

6y= 72

divide both sides by 6

6y/6 = 72/6

y = 12

so therefore, the perimeter of ∆PQR is

9 + 12 + 14

= 35ft

ASAP!!! PLEASE help me solve this question! No nonsense answers, and solve with full solutions.

Answers

Answer:

Option (4)

Step-by-step explanation:

By the theorem of inscribed angles and the intercepted arc,

"In a circle, angles subtended by the same arc always measure the same and the arc measures the double of the inscribed angle."

If an inscribed angle in a circle measures 75° then all inscribed angles by the same arc will measure 75°.

In addition to this, measure of arc subtended by these inscribed angle will measure double of the inscribed angle (150°)

Therefore, Option (4) will be the answer.

(x^2+12x+36)-(x-1)^2
x^2-4xy+4y^2
16x^2-8x+1
x^2+4x+4
4x^2+12xy+9y^2
x^2-8x-16
4x^2-16
x^2-1
x^2+6x+9
9x^2-25y^4
16x^2-8x+1

Answers

Answer:

Hey I'm sorry I didn't get to answer your question it's just that I need the points because I don't have enough to get help with my question. I hope you get the answer that you need for you question. Good Luck :)

Step-by-step explanation:

Please give me the correct answer

Answers

Answer:

Height = 15

Step-by-step explanation:

[tex]Volume = 392.5\\r = 5\\h =?\\\\V= \frac{1}{3} \pi r^2 h\\\\392.5 = \frac{1}{3} \times 3.14 \times 5^2 \times h\\\\392.5 = \frac{78.5h}{3} \\\\392.5 = 26.16h\\\\\frac{392.5}{26.16} =\frac{26.16h}{26.16} \\\\h = 15.00[/tex]

Answer:

h=15 in

Step-by-step explanation:

V=πr²(h/3)

h=3v/πr²

h=[3(392.5)]/[3.14(5)²]

h= 15 in

The sum of the digits of a two digit number is 10 when the dishes are reversed the number increases by 18 find the original number

Answers

Answer:

Step-by-step explanation:

Hello, we can write this number ab where a and b are integer betwen 0 and 9.

For instance, 54, a = 5, b = 4

And then, we can say ab = 10 * a + b.

For instance, 54  = 50 + 4 = 5*10 + 4.

The sum of the digits of a two digit number is 10.

a +  b = 10

When the dishes are reversed the number increases by 18.

10b + a = 18 + 10a + b

9b = 18 + 9a

b = 2 + a

We replace in the first equation to get.

a + 2 + a = 10

2a = 10 -2 = 8

a = 4

and then, b = 6

So, the number is 46.

Thank you

complete the first 4 steps for graphing the quadratic function given.
y= -x^2 -4x -3

ty<3

Answers

To be honest, I'm not sure which four steps your teacher is referring to. However, I'll show you one way to graph this.

A graph is simply a collection of points. Often those points are connected in some way (though they don't necessarily have to be) to form a curve.

Each point is of the form (x,y). To get each point, we pick random x values and determine their paired y value counterpart.

For example, if we pick x = -3, then,

y= -x^2 -4x -3

y= -(-3)^2 -4(-3) -3

y = -9 - 4(-3) - 3

y = -9 + 12 - 3

y = 0

This indicates that (-3, 0) is one point on the curve.

Let's repeat for x = -2

y= -x^2 -4x -3

y= -(-2)^2 -4(-2) -3

y = -4 - 4(-2) - 3

y = -4 + 8 - 3

y = 1

So (-2, 1) is another point on the curve.

Repeat this process as many times as you want. You should do at least 3 or 4 points in my opinion. The more points you generate, the more accurate the curve. After generating the points, you'll plot them all on the same xy grid. Then finally draw a curve through all of the points as shown below.

I used GeoGebra to make the graph.

help me with the picture please

Answers

Answer:  115

=====================================================

Explanation:

Refer to the diagram below. I've drawn diagonal that slopes upward. This diagonal cuts the quadrilateral into two triangles: One is equilateral and the other is isosceles.

The equilateral triangle marked in blue has all three angles 60 degrees each.

Note that the 60 and y angles combined to form 130, so,

60+y = 130

y = 130-60

y = 70

Then focus on the isosceles triangle (angles y, w and w). These three interior angles must add to 180

y+w+w = 180

70+2w = 180

2w = 180-70

2w = 110

w = 110/2

w = 55

This adds onto its adjacent neighbor of 60 to get w+60 = 55+60 = 115 degrees which is the value of x.

The function f(x) = -(x - 3)2 + 9 can be used to represent the area of a rectangle with a perimeter of 12 units, as a
function of the length of the rectangle, x. What is the maximum area of the rectangle?
3 square units
6 square units
9 square units
12 square units

Answers

Answer:

  9 square units

Step-by-step explanation:

The function f(x) describes a parabola opening downward, with a vertex at (3, 9). The maximum value of f(x) is found at the vertex, where it is f(3) = 9.

The maximum area is 9 square units.

Answer:

9 c

Step-by-step explanation:

Pls pls help me! What is it

Answers

Answer:

H

Step-by-step explanation:

Jackie

[tex] \sqrt{9} + \sqrt{25} = 3 + 5 = 8[/tex]

Jesse

[tex] \sqrt{9 + 25} = \sqrt{34} [/tex]

Taking the difference, we get the answer to be

[tex]8 - \sqrt{34} [/tex]

Find the value of x.

Answers

Answer:

[tex]\textsf{x=22.5}[/tex]

Step-by-step explanation:

[tex]\textsf{According to intersecting tangent - secant theorem,}[/tex]

[tex]\textsf{x=1/2[(4x+5)-50]}[/tex]

[tex]\textsf{2(x)=4x+5-50}[/tex]

[tex]\textsf{2x=4x-45}[/tex]

[tex]\textsf{2x-4x=-45}[/tex]

[tex]\textsf{-2x=-45}[/tex]

[tex]\textsf{x= -45/-2}[/tex]

[tex]\textsf{x=22.5}[/tex]

[tex]\textsf{OAmalOHopeO}[/tex]

PLEASE HELP, GIVING 55 POINTS IF YOU ANSWER (08.03/08.05 MC) A pair of equations is shown below: y = 3x − 5 y = 6x − 8 Part A: Explain how you will solve the pair of equations by substitution or elimination. Show all the steps and write the solution. (5 points) Part B: If the two equations are graphed, at what point will the lines representing the two equations intersect? Explain your answer. (5 points)

Answers

Answer:

  A. (1, -2)

  B. the lines intersect at the solution point: (1, -2).

Step-by-step explanation:

A. The equations can be solve by substitution by using the y-expression provided by one of them to substitute for y in the other.

This gives ...

  3x -5 = 6x -8

Adding 8-3x to both sides, we get ...

  3 = 3x

Dividing both sides by 3 gives ...

  1 = x

Substituting this value into the first equation, we can find y:

  y = 3(1) -5 = -2

The solution is (x, y) = (1, -2).

__

B. The lines intersect at the solution point, the point that satisfies both equations simultaneously. That point is (1, -2).

PLEASE HELP MEEE How can a company use a scatter plot to make future sale decisions

Answers

Answer:

by tracking data of how much money was made on one product in a certain amount of time

Step-by-step explanation:

A combination lock uses three numbers between 1 and 46 with​ repetition, and they must be selected in the correct sequence. Is the name of​ "combination lock"​ appropriate? Why or why​ not? Choose the correct answer below. A. ​No, because the multiplication counting rule would be used to determine the total number of combinations. B. ​Yes, because the combinations rule would be used to determine the total number of combinations. C. ​No, because factorials would be used to determine the total number of combinations. D. ​No, because the permutations rule would be used to determine the total number of combinations.

Answers

The correct answer is D. ​No because the permutations rule would be used to determine the total number of combinations.

Explanation:

The difference between a combination and a permutation is that in permutations the order is considered. This applies to the numbers in a lock because these need to be in order. Therefore, to analyze the permutations in a lock, the rule for permutations should be used. This includes the general formula P (n,r) =[tex]\frac{n!}{(n-r) !}[/tex]; in this, n is the number of objects and r refers to the objects used in a permutation. Thus, the term "combination" is inappropriate because this is a permutation, and the permutation rule should be used.

Identify the rate of change and term 0
1. 3, 5, 7, 9, 11, 13, 15....​

Answers

Answer:

-1

mark me brainliest

A sandwich shop offers a choice of 5 types of bread, 4 types of meat, and 3 types of cheese. How many different sandwiches could be made with 1 type of bread, 1 type of meat, and 1 type of cheese?

Answers

Answer:

12 type of bread 1 type of meat, 1 type of cheese and 1 type of bread.☺️☺️

I need help please so if you could help that would be nice. Also i will make brainliest

Answers

No it is not a function. You have to be able to write y=mx+b

The Tama, Japan, monorail carries 92,700 riders

each day. If the monorail usually carries

5,150 riders per hour, how many hours does

the monorail run each day?

Answers

Answer:

The number of hours monorail run each day is 18.

Step-by-step explanation:

The total number of riders the monorail carry each day is:

N = 92700.

The number of riders the monorail carry per hour is:

n = 5150.

Compute the number of hours the monorail run each day as follows:

[tex]\text{Number of hours the monorail run each day}=\frac{N}{n}[/tex]

                                                                     [tex]=\frac{92700}{5150}\\\\=18[/tex]

Thus, the number of hours monorail run each day is 18.

46°
Х
>
X =
degrees
Hshshshsus

Answers

An exterior angle of a triangle is equal to the sum of its interior opposite angles.

⇛ x = 90° + 46°

⇛ x = 136°

Evaluate the following expression.
28 – 10 – 15 = 3 =
and this is the order of operations

Answers

Answer:

28 - 10 - 15 - 3

=> 18 - 15 - 3

=> 3 - 3

=> 0

Another way:

=> 28 - 10 - 15 - 3

=> 28 - 25 - 3

=> 28 - 28

=> 0

A goat is grazing outside of a rectangular barn that has dimensions 20 ft by 30 ft.
He's tied to a corner of the barn with a 12 ft rope. Find the grazing area of the goat.

Answers

Answer:

??????????????????????????????

65. Given a segment with endpoints A and C and midpoint. If A(5, 8), and M(-3,2). Find the
location of C.

Answers

Answer:

C(-11,-4)

Step-by-step explanation:

Other Questions
You plan to buy a $127,242 house. You have $30,313 to use as the down payment. The bank offers to loan you the remainder at 18% nominal interest compounded monthly. The term of the loan is 20 years. What is your equal monthly loan payment A stock has a beta of 1.28, the expected return on the market is 12 percent, and the risk-free rate is 4.5 percent. What must the expected return on this stock be? (Do not round intermediate calculations and enter your answer as a percent rounded to 2 decimal places, e.g., 32.16.) For a transverse wave, what is a wavefront?A a line joining all points on the same crest of a waveB a line showing the displacement of a waveC the energy content of a waveD the first part of a wave to reach a point Is the following question a strong research topic? What effects has The Red Cross had on the world?A. YesB. No, it is too broadC. No, it is too subjective.D. No, it is too narrow create a line that is perpendicular to AB and passes through C. you can use the tools available in geogebra to create perpendicular lines for this construction display the measurement of the angle of intersection between the two lines??? what is a irrational number between 9.5 and 9.7 Which Expression is equivalent to -2(x + 6) 2x + (-12) 2x + 12 2x + 6 PLEASE HELP ME WITH THIS QUESTION PLEASE HELP!!! WILL MARK BRAINLIEST!!What role did economic issues play in the Founding Fathers decisions about how to structure the United States government, including what powers the government would hold over trade and commerce?Please give a detailed explanation preferably paraphrased thank you! Which of the following best explains why community is an influential factor in shaping your relationship, attitude, and habits with food?A. If you do not have easily accessible places to buy healthy food, you're less likely to buy it.B. Not only does the media tell us what to eat, it tells us what not to eat, as well.C. When you're out of the house, such as at a park, you eat what is available if you are hungry.D. You will eat nearly all of your meals with your family, especially when you are young. Have you ever noticed that aside from zoos, you don't see kangaroos, wombats, and koala bears outside of Australia? This is because there is a trench that is a "biogeographical transition zone between Asian and Australian flora and fauna". What is the name of that trench? Please answer ASAP! Type your response in the box. Jack and Mia are playing a game with pick-up sticks. Mia places a pile of 100 pick-up sticks on the table. Forty of the sticks are black, and the rest are brown. She randomly splits all the sticks into two pilesone on Jacks left and one on his right. Mia tells Jack that there are 44 brown pick-up sticks in the pile on his right. Jack looks at the pile of pick-up sticks on his left and estimates that it contains 44 sticks in all. Now Mia blind folds Jack and asks him to choose a stick at random. Jack knows that if he selects a black pick-up stick, Mia will treat him to dinner at his favorite restaurant. If he picks a brown one, then he will treat Mia to dinner at her favorite restaurant. Mia gives Jack three options for selecting: Choose randomly from the pile on the left. Choose randomly from the pile on the right. Push the piles back together and choose randomly from the entire pile. Which option should Jack choose so that Mia treats him to dinner at his favorite restaurant? Explain your answer. It's impossible to pick out a drunk driver until it is too late.A. TrueB. False Your car gets 15 miles per gallon and your friend's car averages 25 mpg. You decidehead off to St. George Island on vacation, 361 miles away. If gas costs $2.79/gallon and you decide to split thegas costs, how much money will you save by driving your friend's car? Marta esta poniendo sus libros en una estantera. Le faltan 7 libros para poder poner 12 en cada estante; sin embargo, si pone 10 libros en cada estante, se quedan 5 libros sin poner. Cuantos es antes tiene la estantera? given the mapping f:x-7x-2, determine f(2) am gonna cry plz answer Which is not an inter molecular force? Select one: a. Inter atomic bonds b. van der waals forces c. hydrogen bonds Can someone help me out with this one? + add an explanation on how you got the answer? Thanks! What is the answer? Which most accurately describes how the Agricultural Revolution impacted the evolution of human society?